LSAT 57 – Section 2 – Question 17

You need a full course to see this video. Enroll now and get started in less than a minute.

Target time: 2:03

This is question data from the 7Sage LSAT Scorer. You can score your LSATs, track your results, and analyze your performance with pretty charts and vital statistics - all with a Free Account ← sign up in less than 10 seconds

Question
QuickView
Type Tags Answer
Choices
Curve Question
Difficulty
Psg/Game/S
Difficulty
Explanation
PT57 S2 Q17
+LR
Weaken +Weak
A
15%
162
B
3%
159
C
5%
159
D
21%
161
E
56%
167
154
163
171
+Hardest 144.702 +SubsectionEasier

Examine (D) closely to understand what it's actually saying.

According to the passage: The BWR makes enough money from sales of its anthologies to cover most operating expenses.

Say its "operating expenses" are $105. That's internet, rent, electricity, etc. costs. Further, say, income from "sales of anthologies" is $100. That means "most" ($100) of the $105 is covered by income from anthology sales.

We still have $5 left that's not covered. Let's give this $5 a name. How about "operating expenses not covered by income from anthology sales?" Because that's what it is. 

Now look at (D).

The BWR depends on donations to cover most operating expenses not covered by income from anthology sales.

In other words, (D) says: The BWR depends on donations to cover most of $5. So... what, like $3.67? 

What the hell does that have to do with anything?

(E) is right. Drawing relevant distinctions is how we weaken arguments by analogy. (E) draws the distinction between "your" magazine's anthology and the BWR's anthology. We just kind of presumed that the BWR's anthology would contain only a reprint of stuff already previously printed, which brings up the question "why in the world would anyone spend money on the anthology when it contains only stuff that I already have in separate editions of the magazine?" (E) tells wrecks that presumption. There's new stuff in the BWR's magazine. That's maybe (likely) why people are paying money to buy it. That means the original suggestion in the passage for "your" magazine to just do a reprint will result in an anthology very much unlike the BRW's anthology.

(A) is also incorrect. Parse out what (A) is saying. Like in (D) we have this concept of "most operating expenses". (A) tells us the money that covers "most operating expenses" isn't donation money. Okay, so what about the remaining expenses? Is that donation covered? We don't know. So could it be that the BWR and your magazine still depend on donations? Yes.

That's even besides the point. The point is what I said in (E) about how you weaken arguments by analogy.

Take PrepTest

Review Results

Leave a Reply